5
$\begingroup$

I am trying to perform multinomial logistic regression on my data which is as below(just the header). enter image description here

"category" is my target variable and all other variables are independent variables. category has values like 1,2,3,4,5,6,7,8,9. My main motive is to predict category from independent variables. Here is summary of my data set. summary

I used following command to perform multinomial logistic regression:

> mod=multinom(category~hlen+iplen+ipttl+iptype+tcpsport+tcpdport+tcpsec+tcpack+tcpwindow+tcpchksum+date_time, data=train)

command

I got the following output but i don't know how to interpret it? enter image description here What should be the starting point or is there other way to do so? (For example I know how to interpret linear and logistic regression output to create regression equations)

Please right click any image and select view to see it clearly. Thank you,

$\endgroup$
1
  • 1
    $\begingroup$ Please replace the images with the actual text. $\endgroup$ Dec 17, 2012 at 16:28

1 Answer 1

2
$\begingroup$

Multinomial logistic regression works like a series of logistic regressions, each one comparing two levels of your dependant variable. Here, category 1 is the reference category.

For example, consider the case where you only have values where category is 1 or 5. (Recode that to 0 and 1, so that you can perform logistic regression.) The coefficients of line 5 of your output represent the variable coefficients such a logistic regression would yield.

$\endgroup$
3
  • $\begingroup$ Thanks mzuba, So is it correct to write logit for category 2 as(considering intercept as "ß0", hlen coefficient as "ß1",iplen coefficient as "ß2" and so on....): logit(p)=ß0+ß1(hlen)+ß3(iplen)+ß4(ipttl)-ß5(iptype)-ß6(tcpsport)-ß7(tcpdport)-ß8(tcpsec)-ß9(tcpack)-ß10(tcpwindow)+ß11(tcpchksum)+ß12(date_time) The values of ß1,ß2,ß3...so on, I can get from above output? $\endgroup$
    – user16603
    Dec 3, 2012 at 17:44
  • $\begingroup$ log[p(category=2)/p(category=1)]=ß0+ß1(hlen)+ß3(iplen)+ß4(ipttl)-ß5(iptype)-ß6(tcpsport)-ß7(tcpdport)-ß8‌​(tcpsec)-ß9(tcpack)-ß10(tcpwindow)+ß11(tcpchksum)+ß12(date_time) The values of ß1,ß2,ß3...so on, I can get from above output? p=probability $\endgroup$
    – user16603
    Dec 3, 2012 at 21:25
  • $\begingroup$ Yes. The intercept values is you B1, B2 and so on values $\endgroup$
    – user107121
    Mar 1, 2016 at 23:55

Your Answer

By clicking “Post Your Answer”, you agree to our terms of service and acknowledge you have read our privacy policy.